2021 AMC 10A Spring Exam Solutions
Scroll down to view professional video solutions and written solutions curated by LIVE, by Po-Shen Loh, print PDF solutions, view answer key, or:
All of the real AMC 8 and AMC 10 problems in our complete solution collection are used with official permission of the Mathematical Association of America (MAA).
Want to learn professionally through interactive video classes?
1.
What is the value of
Solution:
Thus, D is the correct answer.
2.
Portia's high school has times as many students as Lara's high school. The two high schools have a total of students. How many students does Portia's high school have?
Solution:
Let be the number of students in Lara's high school. Then Portia's high school has students.
Therefore, Then
Thus, C is the correct answer.
3.
The sum of two natural numbers is One of the two numbers is divisible by If the units digit of that number is erased, the other number is obtained. What is the difference of these two numbers?
Video solution:
Click to load, then click again to play
Written solution:
Let and be the two numbers. WLOG, let be divisible by Then the units digit of is
If we erase the units digit, then we are essentially dividing by The problem statement also gives us that
Therefore, Then
Thus, D is the correct answer.
4.
A cart rolls down a hill, travelling inches the first second and accelerating so that during each successive -second time interval, it travels inches more than during the previous -second interval. The cart takes seconds to reach the bottom of the hill. How far, in inches, does it travel?
Video solution:
Click to load, then click again to play
Written solution:
The distance travelled every second forms an arithmetic sequence:
The sum of an arithmetic sequence is \text{# of terms} \cdot \dfrac{\text{first + last}}{2}. We know the number of terms is and the first term is The last term is
Plugging these values into the expression yields
Thus, D is the correct answer.
5.
The quiz scores of a class with students have a mean of The mean of a collection of of these quiz scores is What is the mean of the remaining quiz scores in terms of
Video solution:
Click to load, then click again to play
Written solution:
The sum of the scores of everyone in the class is The sum of the scores in the collection of is
This means that the sum of the scores of everyone not in the collection is There are also people not in the collection. Therefore, the average is
Thus, B is the correct answer.
6.
Chantal and Jean start hiking from a trailhead toward a fire tower. Jean is wearing a heavy backpack and walks slower. Chantal starts walking at miles per hour. Halfway to the tower, the trail becomes really steep, and Chantal slows down to miles per hour. After reaching the tower, she immediately turns around and descends the steep part of the trail at miles per hour. She meets Jean at the halfway point. What was Jean's average speed, in miles per hour, until they meet?
Video solution:
Click to load, then click again to play
Written solution:
Let be the distance from the fire tower, where
Then Chantal hiked for hours.
If Jean travelled miles in hours, then his speed was miles per hour.
Thus, A is the correct answer.
7.
Tom has a collection of snakes, of which are purple and of which are happy. He observes that
• all of his happy snakes can add,
• none of his purple snakes can subtract, and
• all of his snakes that can't subtract also can't add.
Which of these conclusions can be drawn about Tom's snakes?
Purple snakes can add.
Purple snakes are happy.
Snakes that can add are purple.
Happy snakes are not purple.
Happy snakes can't subtract.
Video solution:
Click to load, then click again to play
Written solution:
Note that the third condition ensures that purple snakes can't add.
We also know that all happy snakes can add, which means that happy snakes can't be purple as well.
Thus, D is the correct answer.
8.
When a student multiplied the number by the repeating decimal, where and are digits, he did not notice the notation and just multiplied times Later he found that his answer is less than the correct answer. What is the -digit number
Video solution:
Click to load, then click again to play
Written solution:
We can express as an infinite geometric sum: We can therefore use the formula for the sum of a geometric sum: We also know that
Then
Thus, E is the correct answer.
9.
What is the least possible value of for real numbers and
Video solution:
Click to load, then click again to play
Written solution:
Expanding, we get Note that every square must be non-negative. Therefore, the minimum value is when all the terms except are making the sum
This is attainable when
Thus, D is the correct answer.
10.
Which of the following is equivalent to
Video solution:
Click to load, then click again to play
Written solution:
Notice that if we multiply by we end up with a bunch of difference of squares.
This ends up giving us a final value of
Thus, C is the correct answer.
11.
For which of the following integers is the base- number not divisible by
Video solution:
Click to load, then click again to play
Written solution:
We can express this expression in base using the definition of bases:
For this to be divisible by either or must be divisible by
The only answer choice that satisfies neither of these conditions is
Thus, E is the correct answer.
12.
Two right circular cones with vertices facing down as shown in the figure below contains the same amount of liquid. The radii of the tops of the liquid surfaces are cm and cm. Into each cone is dropped a spherical marble of radius cm, which sinks to the bottom and is completely submerged without spilling any liquid. What is the ratio of the rise of the liquid level in the narrow cone to the rise of the liquid level in the wide cone?
Video solution:
Click to load, then click again to play
Written solution:
Let the heights of the narrow wide cones be and respectively.
Then which gives us Also note that and must remain constant due to similar triangles.
After the marble is added to each cone, let be the radius of the narrow cone and be that of the wide cone.
Then the height of the narrow cone is and that of the wide cone is due to similar triangles.
Equating the final volumes, we get which gives us We know that so this equation gives us that
The desired ratio is
Thus, E is the correct answer.
13.
What is the volume of tetrahedron with edge lengths and
Video solution:
Click to load, then click again to play
Written solution:
Analyzing the side lengths, we can realize that are both right triangles.
Therefore, we can treat as the base of the tetrahedron and as the altitude.
With this setup, we can use the formula for the volume of a pyramid, yielding
Thus, C is the correct answer.
14.
All the roots of the polynomial are positive integers, possibly repeated. What is the value of
Video solution:
Click to load, then click again to play
Written solution:
By Vieta's formulas, we get that the product of the roots is and that their sum is
Given that all the roots are positive integers, we can see that the roots are
The function is therefore just
Calculating just the term, we get
Thus, A is the correct answer.
15.
Values for and are to be selected from without replacement (i.e. no two letters have the same value). How many ways are there to make such choices so that the two curves and intersect?
(The order in which the curves are listed does not matter; for example, the choices is considered the same as the choices )
Video solution:
Click to load, then click again to play
Written solution:
Setting the equations equal to each other, we get since squares are non-negative.
This means and must both have the same sign.
If we choose two distinct values for and there are ways to arrange them such that the numerator and denominator both have the same sign.
We have to divide by however, since the two curves are not considered distinct.
Therefore, the total number of tuples is
Thus, C is the correct answer.
16.
In the following list of numbers, the integer appears times in the list for What is the median of the numbers in this list?
Solution:
The total number of numbers in the list is
We want to find the median such that is near
Multiplying by we want near Note that
Plugging in yields
which shows that is our desired median ( is the th and th number).
Thus, C is the correct answer.
17.
Trapezoid has and Let be the intersection of the diagonals and and let be the midpoint of
Given that the length of can be written in the form where and are positive integers and is not divisible by the square of any prime. What is
Video solution:
Click to load, then click again to play
Written solution:
First, we can show that
Let Then since is isosceles.
Since we get that Then since and are right triangles, they are similar.
Using this fact, we get From this, we get that
We also get that from parallel sides and vertical angles.
Therefore Solving, we get and
Using the Pythagorean Theorem on we get that
Thus, D is the correct answer.
18.
Let be a function defined on the set of positive rational numbers with the property that for all positive rational numbers and Suppose that also has the property that for every prime number For which of the following numbers is
Video solution:
Click to load, then click again to play
Written solution:
Note that for any number of the form where is prime, This can be seen by applying the function property multiple times.
Also note that which gives us
We can know calculate each answer choice one-by-one.
Thus, E is the correct answer.
19.
The area of the region bounded by the graph of is where and are integers. What is
Solution:
We can case on the signs of and
Case
Substituting and simplifying, we get This is a circle with radius centered at
Case
As above, we get This is a circle with radius centered at
Case
Again, we get This is a circle with radius centered at
Case
Finally, we get This is a circle with radius centered at
These circles form the following graph:
The semicircles form full two circle with radius for a total area of
The middle square also contributes to the total area. This means that the area of the region is
Thus, E is the correct answer.
20.
In how many ways can the sequence be rearranged so that no three consecutive terms are increasing and no three consecutive terms are decreasing?
Video solution:
Click to load, then click again to play
Written solution:
Note that all the sequences are symmetric about is a valid sequence if and only if is a valid sequence.
Therefore, we can just count all the sequences that begin with and
•
•
•
•
•
•
•
•
•
•
•
•
•
•
•
•
This shows that there are valid sequences starting with the above numbers. Doubling this yields the total number of sequences,
Thus, D is the correct answer.
21.
Let be an equiangular hexagon. The lines and determine a triangle with area and the lines and determine a triangle with area The perimeter of hexagon can be expressed as where and are positive integers and is not divisible by the square of any prime. What is
Solution:
Let and be the points at and respectively.
Since is equiangular, all of its interior angles are
This means that the exterior angles are all making all the small triangles equilateral triangles. This also shows that and are also equilateral.
We know from the problem statement that Solving gives us that and
Finally, the perimeter of is
With this, we have
Thus, C is the correct answer.
22.
Hiram's algebra notes are pages long and are printed on sheets of paper; the first sheet contains pages and the second sheet contains pages and and so on. One day he leaves his notes on the table before leaving for lunch, and his roommate decides to borrow some pages from the middle of the notes. When Hiram comes back, he discovers that his roommate has taken a consecutive set of sheets from the notes and that the average (mean) of the page numbers on all remaining sheets is exactly How many sheets were borrowed?
Video solution:
Click to load, then click again to play
Written solution:
Let the sheets the roommate took be through This is the same as taking pages through
The sum of the pages taken is The sum of the pages remaining is The sum of these equals the sum of all the pages,
Therefore
Then
We can set and to get and This yields
The desired answer is
Thus, B is the correct answer.
23.
Frieda the frog begins a sequence of hops on a grid of squares, moving one square on each hop and choosing at random the direction of each hop-up, down, left, or right. She does not hop diagonally. When the direction of a hop would take Frieda off the grid, she "wraps around" and jumps to the opposite edge. For example if Frieda begins in the center square and makes two hops "up", the first hop would place her in the top row middle square, and the second hop would cause Frieda to jump to the opposite edge, landing in the bottom row middle square.
Suppose Frieda starts from the center square, makes at most four hops at random, and stops hopping if she lands on a corner square. What is the probability that she reaches a corner square on one of the four hops?
Solution:
Let denote the center, an edge, and a corner.
The only ways Frieda can reach a corner in or less moves are
We have to find the probability of each of these cases happening.
Case
On the first hop, Frieda necessarily moves to an From there, Frieda has a chance of going to a This gives us a probability of for this case.
Case
Again, Frieda moves to an on her first move. On the second move, however, she has a chance of wrapping around to another Then there is a chance of going to a from the This is a probability of
Case
Using the same logic as above, this case yields a probability of
Case $ : EMEC
Finally, this case has a probability of
Adding the probabilities together yields a total probability of
Thus, D is the correct answer.
24.
The interior of a quadrilateral is bounded by the graphs of and where is a positive real number. What is the area of this region in terms of valid for all
Solution:
Note that each of the equations yields two parallel lines.
results in the two lines and Both of these lines have a slope of
Similarly, results in the lines and These lines have slope
Note that each pair of lines is perpendicular to the other pair of lines. This shows that the equations form a rectangle.
Recall that the formula for the distance between two parallel lines is
Using this formula, we get that the distance between the first pair of lines is Similarly, the distance between the second pair of lines is
These are the side lengths of the rectangle. Multiplying yields the area
Thus, D is the correct answer.
25.
How many ways are there to place indistinguishable red chips, indistinguishable blue chips, and indistinguishable green chips in the squares of a grid so that no two chips of the same color are directly adjacent to each other, either vertically or horizontally?
Solution:
Let the colors be and Note that we can assign the colors to them in ways, so we have to multiply by at the end.
Let be in the center of the grid.
The other s have to either be along the diagonal or on the same side.
The first scenario can happen in ways since there are diagonals. The second has ways since there are sides.
Either way, the positions of the s and s is fixed.
This is a total of ways to arrange the and s. This gives us a total of configurations.
Thus, E is the correct answer.